Đến nội dung

Kamii0909 nội dung

Có 155 mục bởi Kamii0909 (Tìm giới hạn từ 25-04-2020)



Sắp theo                Sắp xếp  

#662811 Có số nguyên dương n nào thỏa mãn ${3^n} + 2003\,\,...

Đã gửi bởi Kamii0909 on 23-11-2016 - 14:42 trong Số học

Ta sẽ chứng minh không tồn tại.
Thật vậy ta phải có
$8|3^n+3$
Với $n=2k,8|3^n-1$
Với $n=2k+1,8|3^n-3$
Từ đó có đpcm.



#669234 Chứng minh đường tròn đi qua 1 điểm cố định

Đã gửi bởi Kamii0909 on 21-01-2017 - 21:07 trong Hình học

Cho tứ giác $ABCD$ cố định và 1 điểm $P$ chuyển động trên đường $AC$. Giả sử đường tròn ngoại tiếp tam giác $APD$ cắt tia $AD$ tại $E$, đường tròn ngoại tiếp tam giác $APB$ cắt tai $AD$ tại $F$. Chứng minh rằng khi đó đường tròn ngoại tiếp tam giác $AEF$ luôn đi qua 1 điểm cố định.
[hide] Với cấu hình khá là đẹp thế này không biết liệu nó đã xuất hiện ở đâu chưa? [\hide]



#657053 Chứng minh rằng: $\sum \frac{1}{\sqrt{3+a}}\le...

Đã gửi bởi Kamii0909 on 07-10-2016 - 22:04 trong Bất đẳng thức và cực trị

$\sum \frac{1}{\sqrt{3+a}}\leq \sqrt{3\left ( \sum \frac{1}{a+3 } \right )}$

Ta sẽ chứng minh $\sum \frac{1}{3+a}\leq \frac{3}{4}$

Đổi biến $\left ( a,b,c \right )= \left ( \frac{x}{y},\frac{y}{z},\frac{z}{x} \right )$

Đpcm $\Leftrightarrow \sum \frac{y}{x+3y}\leq \frac{3}{4}\Leftrightarrow \sum \frac{3y}{x+3y}\leq \frac{9}{4}\Leftrightarrow \sum \frac{x}{x+3y}\geq \frac{3}{4}$

Điều này luôn đúng do $\sum \frac{x}{x+3y}= \sum \frac{x^{2}}{x^{2}+3xy}\geq \frac{\left ( \sum x \right )^{2}}{\sum x^{2}+3\sum xy}= \frac{\left ( \sum x \right )^{2}}{\left ( \sum x \right )^{2}+\sum xy}\geq \frac{\left ( \sum x \right )^{2}}{\left ( \sum x \right )^{2}+\frac{1}{3}\left ( \sum x \right )^{2}}=\frac{3}{4}$




#670560 Chứng minh rằng: $\left | \frac{a^{3}-b^{...

Đã gửi bởi Kamii0909 on 06-02-2017 - 19:24 trong Bất đẳng thức và cực trị

Cho a, b, c là các số thực dương. Chứng minh rằng:
$\left | \frac{a^{3}-b^{3}}{a+b}+\frac{b^{3}-c^{3}}{b+c}+\frac{c^{3}-a^{3}}{c+a} \right |\leqslant \frac{1}{4}\left [(a-b)^{2}+(b-c)^{2}+(c-a)^{2} \right ]$

Hằng số tốt nhất cho bất đẳng thức này khá xấu và có thể tìm bằng dồn biến toàn miền.
Cho $$a=0,b=2,c=1+ \sqrt{3}+\sqrt{2} \cdot 3^\frac{1}{4}$$ thì $k \geq \sqrt{\dfrac{2 \cdot \sqrt{3}-9}{9}}$



#670339 Chứng minh rằng: $\left | \frac{a^{3}-b^{...

Đã gửi bởi Kamii0909 on 29-01-2017 - 14:07 trong Bất đẳng thức và cực trị

Có vẻ như $\frac{1}{4}$ chưa phải hằng số tốt nhất.
Bình phương lên, điều phải chứng minh tương đương.
$$4\prod(a-b)^2 (ab+bc+ca)^2 \leq \prod (a+b)^2 (\sum a^2-bc)^2$$
Đổi về pqr.
$$ \dfrac{4q^2}{27} [4(p^2-3q)^3 -(2p^3-9pq+27r)^2] \leq (pq-r)^2(p^2-3q)^2 $$
$$L.H.S \leq \dfrac{16q^2(p^2-3q)^3}{27}$$
Ta quy điều phải chứng minh về
$$\dfrac{16q^2(p^2-3q)}{27} \leq (pq-r)^2$$
Có $$pq-r \geq \dfrac{8pq}{9}$$
Thay vào và biến đổi, bất đẳng thức tương đương với
$$q^2(\frac{p^2}{3} +3q) \geq 0$$
Hiển nhiên đúng.



#661255 Chứng minh rằng trong ba số p, q, r có ít nhất hai số bằng nhau.

Đã gửi bởi Kamii0909 on 09-11-2016 - 15:16 trong Số học

Bài này sai đề. Cho $(a,b,c)=(1,2,3)$ không đúng. Theo mình thì p,q,r phải là các số nguyên tố



#661637 Chứng minh rằng trong ba số p, q, r có ít nhất hai số bằng nhau.

Đã gửi bởi Kamii0909 on 12-11-2016 - 14:11 trong Số học

Dễ thấy rằng $a,b,c \geq 1$
Với mọi tính chẵn lẻ của bộ $(a,b,c)$ thì trong 3 số $p,q,r$ luôn có 1 số chẵn.
Giả sử $b^c +a =2$
Từ đó thấy rằng $a=b=1$
Thay xuống $q,r$ ta có $q=r=c+1$
Như vậy ta có đpcm



#661638 Chứng minh rằng $\sum \frac{x^{4}+y^{4...

Đã gửi bởi Kamii0909 on 12-11-2016 - 14:16 trong Số học

Bài này nên đưa vào mục bất đẳng thức chứ nhỉ.
Có thể giải bằng Holder như sau(không hay lắm)
$(x^4 +y^4)^3(1+1) \geq (x^3+y^3)^4$
và $(x^3+y^3)(1+1)(1+1) \geq (x+y)^3$
Từ đó ta có
$\frac{x^4+y^4}{x^3+y^3} \geq \frac{x+y}{2}$
Cộng lại ta cũng có đpcm



#660729 Chứng minh N là trực tâm tam giác

Đã gửi bởi Kamii0909 on 05-11-2016 - 22:40 trong Hình học

Chứng minh cho đầy đủ. Phán phát thế ai chơi Kiên :icon6:

Cần chỗ nào :3



#660680 Chứng minh N là trực tâm tam giác

Đã gửi bởi Kamii0909 on 05-11-2016 - 15:17 trong Hình học

Theo 1 bổ đề quen thuộc thì AX là đối trung tam giác ABC và AN là đối trung tam giác AEF.
Từ tính chất đường đối trung và định lý Thales ta có
$\frac{XB}{XC} = \frac{AB^2}{AC^2} = \frac{AE^2}{AF^2} = \frac{NE}{NF} =\frac{ZE}{ZC}$
Từ đó $ZN//FC$. Tương tự có điều phải chứng minh



#668675 Chứng minh di chuyển trên 1 đườnng cố định

Đã gửi bởi Kamii0909 on 17-01-2017 - 17:10 trong Hình học

Cách của mình. Ta sẽ nhắc lại không chứng minh một số kết quả cơ bản sau.
1. Cho tam giác $ABC$ nội tiếp $(O)$. Tiếp tuyến tại $B,C$ của $(O)$ cắt nhau tại $P$. Khi đó $AP$ là đường đối trung đỉnh $A$ của tam giác $ABC$.
2. Trong tam giác $ABC$, đường đối trung đỉnh $A$ cắt $BC$ tại $D$. Khi đó $\frac{DB}{DC}= \frac{AB^2}{AC^2}$
Để xử lí cho gọn, đẹp trước hết ta đi chứng minh bổ đề sau
$\textbf{Bổ đề}$ Cho tam giác $ABC$. Đường đối trung đỉnh $A$ cắt $BC$ tại $Q$. $O,I,J$ lần lượt tâm đường tròn ngoại tiếp tam giác $ABC,AQB,AQC$. Khi đó $OA$ chia đôi $IJ$.
$\textbf{Chứng minh}$
Kẻ $AH,AM$ là đường cao và là trung tuyến của tam giác $ABC$.
Dễ thấy phép vị tự quay tâm $A$ biến $\Delta AIJ \rightarrow \Delta ABC$ biến $H \rightarrow Q$. Mặt khác cũng có $\angle HAM= \angle QAO$ nên biến $ AM \rightarrow AO$. Lại có $AM$ là trung tuyến trong tam giác $ABC$ nên $AO$ là trung tuyến trong tam giác $AIJ$.

$\textbf{Quay lại bài toán}$
Qua $Q$ kẻ đường thẳng $\parallel BC$ cắt $AB,AC$ tại $E,F$. Gọi $I,J$ là tâm đường tròn ngoại tiếp các tam giác $AQE,AQF$.
Theo bổ đề $OA$ chia đôi $IJ$. Ta sẽ chứng minh rằng $O_{1}I=O_{2}J$.
Thật vậy, có $\Delta AIO_{1} \sim \Delta QEB, \Delta AJO_{2} \sim \Delta QFC$ nên ta thu được biến đổi sau.
$$\dfrac{IO_{1}}{JO_{2}}= \frac{IO_{1}}{IA}\cdot \frac{IA}{JA} \cdot \frac{JA}{JO_{2}} = \frac{EB}{QE}\cdot \frac{AB}{AC}\cdot \frac{FQ}{FC}= \frac{AE^2}{AF^2} \cdot \frac{QF}{QE}=1$$
Vậy ta có điều phải chứng minh.



#663699 Chứng minh CD vuông góc OE

Đã gửi bởi Kamii0909 on 03-12-2016 - 14:48 trong Hình học

Còn 1 cách nữa suy nghĩ thêm đi :D

Cách khác.
Ta nhắc lại không chứng minh bổ đề quen thuộc sau.
Cho tứ giác nội tiếp $ABCD$ có 2 tiếp tuyến tại $B,D$ và $AC$ đồng quy. Khi đó 2 tiếp tuyến tại $A,C$ và $BD$ cũng đồng quy.(Tứ giác điều hòa)
Trở lại bài toán. Gọi $P$ là giao điểm $DC$ và $(O)$.
Khi đó theo bổ đề $EP$ là tiếp tuyến của $(O)$.
Từ đó dễ dàng dẫn đến đpcm.



#663696 Chứng minh CD vuông góc OE

Đã gửi bởi Kamii0909 on 03-12-2016 - 14:18 trong Hình học

Bài toán vẫn đúng trong trường hợp $A$ không là trung điểm $OD$.
Dễ dàng chứng minh được $\Delta OBD \sim \Delta ECB$
Nên $\frac{BO}{CE}=\frac{BD}{BC}=\frac{CO}{CE}$
Kết hợp với $\widehat{ECO}=\widehat{DBO}$ ta thu được $\Delta COE \sim \Delta BDC$ từ đó dễ dàng có đpcm.



#667825 Chứng minh A,F,I thẳng hàng

Đã gửi bởi Kamii0909 on 09-01-2017 - 22:40 trong Hình học

Nó quanh quanh cấu hình bài hình của USAMO 2008



#662540 Chứng minh $MK=MA$

Đã gửi bởi Kamii0909 on 20-11-2016 - 19:52 trong Hình học

Ta có $MK^2=MO^2-R^2=MH^2+HO^2$
$MA^2=MH^2+HA^2$
Gọi $OA$ cắt $BC$ tại $Q$. Khi đó $HA=HQ$
Khi đó ta phải chứng minh
$HO^2-HA^2=R^2$
$\Leftrightarrow (HO-HQ)(HO+HA)=R^2$
$\Leftrightarrow OQ.OA=OB^2$
Đẳng thức cuối là cơ bản.



#662524 Chứng minh $IM$, $IN$ tiếp xúc với $(AMN)$

Đã gửi bởi Kamii0909 on 20-11-2016 - 16:48 trong Hình học

Bài này có khá nhiều cách giải.
1.Dễ thấy $\Delta AMN $ cân.
Gọi giao điểm $(O)$ và $(ADE)$ là $K$.
Khi đó $K$ cũng thuộc $(AMN)$.
Dễ có $AI,KH$ cắt nhau tại 1 điểm trên $(AMN)$ là $X$.
Ta có $\frac{AM}{AN}=\frac{XM}{XN}$ nên AMXN điều hòa.
2. Theo định lý về tâm đẳng phương
$AK,BC,DE$ đồng quy tại $P$.
Gọi $Q$ là giao điểm $AH$ và $KI$.
Do $AO$,$AH$ đẳng giác nên $Q$ thuộc $(AMN)$.
$AQ$ cắt $BC$ tại $Y$.
Dễ có $(PY,BC)=-1$.
Chiếu hàng này lên $(AMN)$ thì $KMQN$ điều hòa.
Tư đó $KQ$ và $AX$ cắt nhau tại $I$ là giao điểm 2 tiếp tuyến của $(AMN)$.



#662553 Chứng minh $I,E,F$ thẳng hàng .

Đã gửi bởi Kamii0909 on 20-11-2016 - 20:54 trong Hình học phẳng

Đề có đúng ko bạn. Mình vẽ nó sai



#662270 Chứng minh $G,G_1,G_2$ thẳng hàng.

Đã gửi bởi Kamii0909 on 17-11-2016 - 22:32 trong Hình học

Ta có $3\overrightarrow{GG_{1}}+3\overrightarrow{GG_{2}}=\sum\overrightarrow{GA_{1}}+\sum\overrightarrow{GA_{2}}=\overrightarrow0$

nên $G,G_{1},G{2}$ thẳng hàng. Cách sử dụng vecto. Bạn tham khảo




#673878 Chứng minh $3(a+b+c)\geq \sqrt{8a^2+1}+\sqrt...

Đã gửi bởi Kamii0909 on 10-03-2017 - 17:40 trong Bất đẳng thức và cực trị

KMTTQ, $a \geq b \geq c$
Khi đó dễ cmr $a - \frac{1}{a} \geq b-\frac{1}{b} \geq c - \frac{1}{c}$
Và $3+ \sqrt{8+ \frac{1}{a^2}} \leq 3+ \sqrt{8+ \frac{1}{b^2}} \leq 3+\sqrt{8+\frac{1}{c^2}}$
Bđt cần cm tương đương với
$\sum \frac{a-\frac{1}{a}}{3+ \sqrt{8+\frac{1}{a^2}}} \geq 0$
Áp dụng bđt Cheybershev kết hợp điều kiện ta có đpcm.



#672386 Chứng minh $\sum \frac{1}{\sqrt{1+a^2...

Đã gửi bởi Kamii0909 on 22-02-2017 - 14:21 trong Bất đẳng thức - Cực trị

Có thể dùng dồn biến. Chú ý rằng với $ab\leq 1$ ta dễ dàng chỉ ra rằng
$$\frac{1}{\sqrt{1+a^2}}+\frac{1}{\sqrt{1+b^2}} \leq \frac{2}{\sqrt{ab+1}}$$. Thay $ab=\frac{1}{c}$ và xét hàm theo $c$.



#663371 CHỨNG MINH THẲNG HÀNG

Đã gửi bởi Kamii0909 on 29-11-2016 - 18:04 trong Hình học

Gọi $AI,BI,CI$ cắt $(ABC)$ tại $M,N,P$.
Dễ dàng chứng minh $I$ là trực tâm tam giác $MNP$.
Bổ đề:Cho tam giác $ABC$ trực tâm $H$ nội tiếp $(O)$. Khi đó $\overrightarrow{OH}=\overrightarrow{OA} +\overrightarrow{OB} + \overrightarrow{OC}$.
Áp dụng bổ đề trên với chú ý $I,O$ là tâm ngoại của $MNP$ và $DEF$ và $\overrightarrow{ID} = \frac{r}{R}.\overrightarrow{OM}$ ta dễ có $\overrightarrow{OI}= \frac{R}{r} \overrightarrow{IH}$
Hệ thức trên chứng tỏ $\overline{O,I,H}$.



#654500 CHỌN ĐỘI TUYỂN HSG QUỐC GIA TỈNH HÒA BÌNH

Đã gửi bởi Kamii0909 on 17-09-2016 - 16:59 trong Thi HSG cấp Tỉnh, Thành phố. Olympic 30-4. Đề thi và kiểm tra đội tuyển các cấp.

biến đổi vế trái:
$ VT=a(x^{2}+z^{2})+b(x^{2}+y^{2})+c(y^{2}+z^{2}) \geq 2(azx+bxy+cyz) $
đến đây áp dụng BĐT chebychev ta có:
$ azx+bxy+cyz \geq \frac{1}{3}(a+b+c)(xy+yz+zx) $
mà theo bất đẳng thức AM-GM ta có: $ a+b+c \geq 3\sqrt[3]{abc} = 3 $
suy ra $ 2(azx+bxy+cyz) \geq 2(xy+yz+zx) $
từ đó ta có đpcm

Hình như đâu có thể Cheybershev được đâu :) Nếu a>=b>=c và y>=x>=z thì bất đẳng thức đó sai rồi mà



#661696 Cho $a^2+b^2+(a-b)^2=c^2+d^2+(c-d)^2$. CMR: $a^4+b^4+(a-b)^4=c...

Đã gửi bởi Kamii0909 on 12-11-2016 - 23:26 trong Số học

Có hằng đẳng thức sau 

$a^4 +b^4 +(a\pm b)^4=2(a^2\pm ab+b^2)^2$

Hoàn toàn dễ dàng chứng minh đẳng thức trên dựa vào đây.




#668867 Cho $\bigtriangleup ABC$ nội tiếp đường tròn tâm $O....

Đã gửi bởi Kamii0909 on 19-01-2017 - 17:31 trong Hình học

Nếu từ điểm M mà hạ MH vuông góc với BC thì đường AH là đường trong bài P17 của thày Hùng ở tạp chí Pi số 1!

Ùi  :D May quá  :closedeyes: Tý thì e lỡ post lời giải bài P.17  :icon6:  :icon6:  :icon6:  




#658525 Chia hết

Đã gửi bởi Kamii0909 on 20-10-2016 - 14:46 trong Số học

Xét số 1111....11 với 1,2,.....,2014 chữ số 1.
Nếu tồn tại 1 số trong các số trên chia hết cho 2013 thì ta có đpcm
Ngược lại ta giả sử không tồn tại thì khi đó sẽ có 2 số trong bảng đồng dư 2013

Lấy hiệu 2 số ta có số $111...100..00=11...11.10^{x}$

Hiển nhiên $gcd(10^{x},2013)=1\Rightarrow 2013|111..11$ 

Vậy luôn tồn tại số thỏa mãn yêu cầu bài toán